-
Notifications
You must be signed in to change notification settings - Fork 2
/
lista13.tex
231 lines (208 loc) · 14.3 KB
/
lista13.tex
1
2
3
4
5
6
7
8
9
10
11
12
13
14
15
16
17
18
19
20
21
22
23
24
25
26
27
28
29
30
31
32
33
34
35
36
37
38
39
40
41
42
43
44
45
46
47
48
49
50
51
52
53
54
55
56
57
58
59
60
61
62
63
64
65
66
67
68
69
70
71
72
73
74
75
76
77
78
79
80
81
82
83
84
85
86
87
88
89
90
91
92
93
94
95
96
97
98
99
100
101
102
103
104
105
106
107
108
109
110
111
112
113
114
115
116
117
118
119
120
121
122
123
124
125
126
127
128
129
130
131
132
133
134
135
136
137
138
139
140
141
142
143
144
145
146
147
148
149
150
151
152
153
154
155
156
157
158
159
160
161
162
163
164
165
166
167
168
169
170
171
172
173
174
175
176
177
178
179
180
181
182
183
184
185
186
187
188
189
190
191
192
193
194
195
196
197
198
199
200
201
202
203
204
205
206
207
208
209
210
211
212
213
214
215
216
217
218
219
220
221
222
223
224
225
226
227
228
229
230
231
% Filename: lista13.tex
%
% This code is part of 'Solutions for MT402, Matrizes'
%
% Description: This file corresponds to the solutions of homework sheet 13.
%
% Created: 16.06.12 08:15:39 AM
% Last Change: 29.06.12 05:39:45 PM
%
% Authors:
% - Raniere Silva (2012): initial version
%
% Copyright (c) 2012 Raniere Silva <r.gaia.cs@gmail.com>
%
% This work is licensed under the Creative Commons Attribution-ShareAlike 3.0 Unported License. To view a copy of this license, visit http://creativecommons.org/licenses/by-sa/3.0/ or send a letter to Creative Commons, 444 Castro Street, Suite 900, Mountain View, California, 94041, USA.
%
% This work is distributed in the hope that it will be useful, but WITHOUT ANY WARRANTY; without even the implied warranty of MERCHANTABILITY or FITNESS FOR A PARTICULAR PURPOSE.
%
\begin{theorem}[Norma matricial induzida\nocite{Meyer:2000:matrix}]
Uma norma vetorial definida em $\mathbb{C}^p$, $p = m, n$, induz uma norma matricial definida em $\mathbb{C}^{m \times n}$ por
\begin{align*}
\| A \| &= \max_{\| x \| = 1} \| A x \|, \forall A \in \mathbb{C}^{m \times n}, x \in \mathbb{C}^{n \times n}.
\end{align*}
Para uma norma matricial induzida vale
\begin{align*}
\| A x \| &\leq \| A \| \| x \|
\end{align*}
e se $A$ for n\~{a}o singular
\begin{align*}
\min_{\| x \| = 1} \| A x \| = 1 / \| A^{-1} \|.
\end{align*}
\end{theorem}
\begin{theorem}[Propriedades da norma-2\nocite{Meyer:2000:matrix}]
Na norma matricial induzida pela norma-2 vetorial vale que
\begin{align*}
\| U^* A V \|_2 = \| A \|_2
\end{align*}
quando $U$ e $V$ s\~{a}o matrizes unit\'{a}rias.
\end{theorem}
\begin{defi}[Posto\nocite{Meyer:2000:matrix}]
O posto de uma matriz $A : m \times n$ \'{e} precisamente a ordem da maior submatriz matriz quadrada n\~{a}o-singular de $A$. Em outras palavras, se $\mathrm{posto}(A) = r$ ent\~{a}o existe pelo menos uma submatriz $r \times r$ n\~{a}o singular em $A$ e n\~{a}o existe nenhuma outra submatriz n\~{a}o-singular de ordem maior que $r$.
\end{defi}
\begin{theorem}[4.2.1 do Watkins\nocite{Watkins:2004:fundamentals}]
Seja $A \in \mathbb{R}^{n \times n}$ com os seguintes valores singulares: $\sigma_1 \geq \sigma_2 \geq \ldots \geq 0$. Ent\~{a}o
\begin{align*}
\| A \|_A = \max_{x \neq 0} \frac{\| A x \|_2}{\| x \|_2} = \sigma_1.
\end{align*}
\end{theorem}
\begin{theorem}[Exerc\'{i}cio 4.2.5 do Watkins\nocite{Watkins:2004:fundamentals}]
Seja $A \in \mathbb{R}^{n \times n}$ com os seguintes valores singulares: $\sigma_1 \geq \sigma_2 \geq \ldots \geq \sigma_n > 0$. Ent\~{a}o
\begin{align*}
\min_{x \neq 0} \frac{\| A x \|_2}{\| x \|_2} = \sigma_n.
\end{align*}
\end{theorem}
\begin{theorem}[4.5.9 do Meyer\nocite{Meyer:2000:matrix}]
Se $A$ e $E$ s\~{a}o matrizes $m \times n$ tal que $E$ possue entradas suficientemente pequenas ent\~{a}o
\begin{align*}
\mathrm{posto}(A + E) \geq \mathrm{posto}(A).
\end{align*}
\end{theorem}
\begin{questions}
\question Considere $A : m \times n$, $\textrm{posto}(A) = r < \min\left\{ m, n \right\}$. Usando a SVD de $A$, mostre que para cada $\epsilon > 0$, existe uma matriz de posto completo $A_\epsilon : m \times n$ tal que $\| A - A_\epsilon \|_2 < \epsilon$.
\begin{parts}
\part[Exerc\'{i}cio 4.2.14 do Watkins\nocite{Watkins:2004:fundamentals}] Escolha convenientemente $\overline{\epsilon}$ para demonstrar a tese para cada $\epsilon > 0$ e manter a ordena\c{c}\~{a}o decrescente dos valores singulares de $A_\epsilon$.
\begin{solution}
Seja $A = U D V^t$, onde $U : m \times m$ e $V : n \times n$ s\~{a}o matrizes ortogonais e $D : m \times n$ \'{e} tal que $d_{ij} = 0$ para $i \neq j$, $d_{ii} = \sigma_i$, $i = 1, 2, \ldots, r$, $r < \min\left\{ m, n \right\}$, $\sigma_1 \geq \sigma_2 \geq \ldots \geq \sigma_r > 0$ e $d_{ii} = 0$, $i = r + 1, \ldots, \min\left\{ m, n \right\}$.
Dado $\epsilon > 0$ construimos $A_\epsilon = U D_\epsilon V^t$ onde $D_\epsilon$ difere de $D$ nas posi\c{c}\~{o}es $d_{ii}$, $i = 1, 2, \ldots, \min\left\{ m, n \right\}$, onde \'{e} acrescido $\bar{\epsilon} < \epsilon$ ao valor de $d_{ii}$.
Ent\~{a}o, temos que
\begin{align*}
\| A - A_\epsilon \|_2 &= \| V^t D U - V^t D_\epsilon U \|_2 \\
&= \| V^t \left( D - D_\epsilon \right) U \|_2 \\
&= \| V^t \tilde{D} U \|_2,
\end{align*}
onde $\tilde{D}$ \'{e} tal que $d_{ij} = 0$ para $i \neq j$, $d_{ii} = \bar{\epsilon}$, $i = 1, 2, \ldots, \min\left\{ m, n \right\}$.
Como sabemos que $\| A \|_2 = \sigma_1$ concluimos que $\| A - A_\epsilon \|_2 = \bar{\epsilon} < \epsilon$.
\end{solution}
\part A matriz $A_\epsilon$ pode ser constru\'{i}da a partir da SVD de $A$, mantendo as matrizes $U$ e $V$ e adicionando $\overline{\epsilon} < \epsilon$ a cada valor singular de $A$ para compor a matriz $D_\epsilon$?
\begin{solution}
Sim, foi esse o m\'{e}todo utilizado no item anterior.
\end{solution}
\end{parts}
\question[Ver p\'{a}gina 271 do Watkins\nocite{Watkins:2004:fundamentals}] Julgue verdadeiro ou falso a afirma\c{c}\~{a}o:
\begin{quote}
O conjunto das matrizes de posto completo \'{e} denso e aberto.
\end{quote}
Demonstre se a afirma\c{c}\~{a}o for verdadeira e d\^{e} um contra-exemplo se for falsa.
\begin{solution}
Em topologia, o conjuntos das matrizes de posto completo \'{e} um subconjunto do $\mathbb{R}^{n \times n}$ aberto e denso, i.e.,
\begin{enumerate}
\item Se $A$ tem posto completo, ent\~{a}o todas as matrizes suficientemente pr\'{o}ximas de $A$ tamb\'{e}m tem posto completo. \label{cond1:denso_aberto}
\item Toda matriz de posto incompleto possue pelo menos uma matriz de posto completo arbitrarirmente pr\'{o}xima dela. \label{cond2:denso_aberto}
\end{enumerate}
No exerc\'{i}cio anterior provamos a condi\c{c}\~{a}o \ref{cond2:denso_aberto}. E a demonstra\c{c}\~{a}o da condi\c{c}\~{a}o \ref{cond1:denso_aberto} \'{e} an\'{a}loga a da condi\c{c}\~{a}o \ref{cond2:denso_aberto} mudando apenas o fato da matriz ter posto completo.
\end{solution}
\question Demonstre que toda matriz $A : m \times n$ \'{e} limite de uma sequencia de matrizes de posto completo.
\begin{solution}
Demonstrar que toda matriz $A : m \times n$ \'{e} limite de uma sequencia de matrizes de posto completo equivale a mostrar que o conjunto das matrizes de posto completo \'{e} denso. No exerc\'{i}cio anterior j\'{a} foi demonstrado que o conjunto das matrizes de posto completo \'{e} denso (e aberto).
\end{solution}
\question Demonstre que $A : m \times n$ com posto $s$ e $B : m \times n$ tal que $\| B - A \|_2 < \sigma_s$, ent\~{a}o $\mathrm{posto}(B) \geq s$.
\begin{solution}
Seja $A = U D V^t$, onde $U : m \times m$ e $V : n \times n$ s\~{a}o matrizes ortogonais e $D : m \times n$ \'{e} tal que $d_{ij} = 0$ para $i \neq j$, $d_{ii} = \sigma_i$, $i = 1, 2, \ldots, s$, $s \leq \min\left\{ m, n \right\}$, $\sigma_1 \geq \sigma_2 \geq \ldots \geq \sigma_s > 0$ e $d_{ii} = 0$, $i = s + 1, \ldots, \min\left\{ m, n \right\}$.
Seja $B = U \tilde{D} V^t$, onde $\tilde{D}$ \'{e} tal que $\tilde{d}_{ij} = 0$ para $i \neq j$, $\tilde{d}_{ii} = \tilde{\sigma}_i$, $i = 1, 2, \ldots, r$, $r \leq \min\left\{ m, n \right\}$, $\tilde{\sigma}_1 \geq \tilde{\sigma}_2 \geq \ldots \geq \tilde{\sigma}_r > 0$ e $d_{ii} = 0$, $i = r + 1, \ldots, \min\left\{ m, n \right\}$.
Ent\~{a}o, temos que
\begin{align*}
\| B - A \|_2 &= \| U \tilde{D} V^t - U D V^t \|_2 \\
&= \| U \left( \tilde{D} - D \right) V^t \|_2.
\end{align*}
Como desejamos que $\| B - A \|_2 < \sigma_s$ temos que $\tilde{\sigma_i} = \sigma_i$ para $i = 1, \ldots, s$, $\tilde{\sigma_i} > 0$ para $i = s + 1, \ldots, r$ e $\tilde{\sigma_i} = 0$ para $i = r + 1, \ldots, \min{m, n}$. Logo, conclui-se que o $\mathrm{posto}(B) \geq s$.
\end{solution}
\question Obtenha a decomposi\c{c}\~{a}o SVD de uma matriz de posto $1$, i.e., se $w : m \times 1$ e $z : n \times 1$ s\~{a}o vetores n\~{a}o nulos, obtenha a decomposi\c{c}\~{a}o SVD de $A = w z^t$.
\begin{solution}
Calculamos
\begin{align*}
A A^t & \left( w z^t \right) \left( w z^t \right)^t \\
&= w z^t z w^t \\
&= \| z \|_2^2 w w^t.
\end{align*}
\end{solution}
\question Considere $A : m \times n$, $m > n$, o vetor $z : m \times n$ e a matriz $B = [A \ z]$. Mostre que $\sigma_{n + 1}(B) \leq \sigma_n(A)$ e $\sigma_1(B) \geq \sigma_1(A)$.
\begin{solution}
Temos que
\begin{align*}
\sigma_1(B) &= \max_{x \neq 0} \frac{\| B x \|_2}{\| x \|_2} \\
&\geq \max_{x \neq 0, x_{n + 1} = 0} \frac{\| B x \|_2}{\| x \|_2} \\
&= \max{x \neq 0} \frac{\| A x \|_2}{\| x \|_2} \\
&= \sigma_1(A).
\end{align*}
% TODO Fazer esse exerc\'{i}cio.
\end{solution}
\question Considere agora $A : m \times n$, $m \geq n$, o vetor $z : n \times n$ e a matriz $B = [A; z^t]$. Mostre que $\sigma_n(B) \geq \sigma_n(A)$ e $\sigma_1(A) \leq \sigma_1(B) \leq \sqrt{\left( \sigma_1(A) \right)^2 + \| z \|_2^2}$.
\begin{solution}
Temos que
\begin{align*}
\sigma_1(B) &= \max_{\| x \|_2 = 1} \frac{\| B x \|_2}{\| x \|_2} \\
&= \max_{\| x \|_2 = 1} \frac{\| [A; z^t] x \|_2}{\| x \|_2}
\end{align*}
% TODO Fazer esse exerc\'{i}cio.
\end{solution}
\question[Ver exerc\'{i}cio 4.2.10 do Watkins\nocite{Watkins:2004:fundamentals}] Considere $A : n \times n$ sim\'{e}trica definida positiva. Demonstre que existe $M$ sim\'{e}trica, tal que $A = M^2$. Sugest\~{a}o: use a SVD do fator de Cholesky de $A$).
\begin{solution}
Como $A$ \'{e} sim\'{e}trica definida positiva existe $G$ tal que $A = G G^t$. Seja $G = U D V^t$, ent\~{a}o
\begin{align*}
A &= G^t G \\
&= \left( U D V^t \right)^t \left( U D V^t \right) \\
&= V D^t U^t U D V^t \\
&= V D^t I D V^t \\
&= V D^t D V^t \\
&= V D D V^t \\
&= V D^2 V^t \\
&= D^2 V V^t \\
&= D^2.
\end{align*}
\end{solution}
\question Valores singulares extremos do produto entre duas matrizes. Se $A : m \times p$ e $B : p \times n$, $q = \min\left\{ m, n \right\}$ e $S = \min\left\{ p, n \right\}$, mostre que
\begin{parts}
\part $\sigma_1(A B) \leq \sigma_1(A) \sigma_1(B)$,
\begin{solution}
Temos que
\begin{align*}
\sigma_1(A B) &= \max_{\| x \| = 1} \frac{\| A B x \|_2}{\| x \|_2} \\
&\leq \max_{\| x \| = 1} \frac{\| A \|_2 \| B x \|_2}{\| x \|_2} \\
&= \| A \|_2 \max_{\| x \| = 1} \frac{\| B x \|_2}{\| x \|_2} \\
&= \| A \|_2 \| B \|_2 \\
&= \sigma_1(A) \sigma_1(B).
\end{align*}
\end{solution}
\part $\sigma_q(A B) \leq \sigma_1(A) \sigma_s(B)$.
\begin{solution}
Temos tamb\'{e}m que
\begin{align*}
\sigma_q(A B) &= \min_{x \neq 0} \frac{\| A B x \|_2}{\| x \|_2} \\
&\leq \min_{x \neq 0} \frac{\| A \|_2 \| B x \|_2}{\| x \|_2} \\
&= \| A \|_2 \min_{x \neq 0} \frac{\| B x \|_2}{\| x \|_2} \\
&= \sigma_1(A) \sigma_s(B).
\end{align*}
\end{solution}
\end{parts}
\question[Exerc\'{i}cio 4.1.17 do Watkins\nocite{Watkins:2004:fundamentals}] Demonstre que uma matriz $A : m \times n$ pode ser expressa como $A = U D V^t$ onde $U : m \times m$ e $V : n \times n$ s\~{a}o ortogonais e $D : m \times n$ \'{e} matriz ``diagonal'' com elementos na diagonal $\sigma_1 \geq \sigma_2 \geq \ldots \geq \sigma_r \geq 0$.
\begin{solution}
Vamos provar por indu\c{c}\~{a}o finita em $r$, o posto da matriz $A$.
\begin{enumerate}
\item Suponha que $A \in \mathbb{R}^{n \times n}$ tem posto $1$. Seja $u_1 \in \mathbb{R}^n$ um vetor em $\mathcal{I}(A)$ tal que $\| u_1 \|_2 = 1$. Ent\~{a}o, como $u_1 \in \mathcal{I}(A)$ verifica-se que existe $x$ tal que
\begin{align*}
A_{\mdot 1} x_1 + A_{\mdot 2} x_2 + \ldots A_{\mdot n} x_n &= u_1
\end{align*}
e como o $\mathrm{posto}(A) = 1$ existe apenas uma coluna linearmente independente em $A$ de modo que reescrevemos a igualdade acima como
\begin{align*}
A_{\mdot 1} \left( c_1 x_1 + c_2 x_2 + \ldots + c_n x_n \right) &= u_1,
\end{align*}
onde $c_1, c_2, \ldots, c_n \in \mathbb{R}$. Logo, verifica-se que cada coluna de $A$ \'{e} um multiplo de $u_1$.
Como $A$ \'{e} uma matriz de posto $1$ ela pode ser escrita na forma $A = \sigma_1 u_1 v_1^t$, onde $v_1 \in \mathbb{R}^m$, $\| v_1 \|_2 = 1$ e $\sigma_1 > 0$.
\item Agora demonstremos que a existe uma matriz ortogonal $U \in \mathbb{R}^{n \times n}$ cuja primeira coluna \'{e} $u_1$.
De maneira similar, existe uma matriz ortogonal $V \in \mathbb{R}^{m \times m}$ cuja primeira coluna \'{e} $v_1$.
E, por fim, que $A = U D V^t$, onde $D \in \mathbb{R}^{n \times m}$ possui um \'{u}nico elemento n\~{a}o zero, $\sigma_1$, na posi\c{c}\~{a}o $(1,1)$.
\item Agora, suponha que $A \in \mathbb{R}^{n \times m}$ possue posto $r > 1$. Seja $v_1$ o vetor unit\'{a}rio tal que $\| A v_1 \|_2 = \max_{\| u \|_2 = 1} \| A u \|_2$. Seja $\sigma_1 = \| A u_1 \|_2 = \| A \|_2$ e $\| u_1 = \sigma_1^{-1} A v_1$. Seja $\tilde{U} \in \mathbb{R}^{n \times n}$ e $\tilde{V} \in \mathbb{R}^{m \times m}$ matrizes ortogonais com primeira coluna $u_1$ e $v_1$, respecitivamente. Seja $\tilde{A} = \tilde{U}^t A \tilde{V}$ tal que $A = \tilde{U} \tilde{A} \tilde{V}^t$. Mostremos que $\tilde{A}$ possue a forma
\begin{align*}
\tilde{A} = \begin{bmatrix}
\sigma_1 & z^t \\
0 & \hat{A}
\end{bmatrix},
\end{align*}
onde $z \in \mathbb{R}^{m -1}$ e $\hat{A} \in \mathbb{R}^{(n - 1) \times (m - 1)}$.
\item Mostre
\item Most
\end{enumerate}
% TODO Fazer esse exerc\'{i}cio.
\end{solution}
\end{questions}